Show that matrices of defined form have inverse of the same same defined form

Click For Summary
The discussion focuses on proving that the inverses of a specific set of 3x3 matrices, defined as [1, a, b; 0, 1, c; 0, 0, 1], maintain the same structural form. The user has successfully demonstrated that the matrices are non-singular and established properties like closure and identity under multiplication. However, they initially struggled to show that the inverses also conform to the defined matrix structure. Another participant pointed out that finding the inverse is straightforward and confirmed that it retains the same form. This confirms that the set of matrices is indeed closed under inversion, supporting the group's properties.
donald17
Messages
6
Reaction score
0

Homework Statement



Given the set of 3x3 matrices of the form: [1, a, b; 0, 1, c; 0, 0, 1], where a, b, and c are any real numbers show that the inverses of these matrices are of the same given form.

Homework Equations



Using elementary row operations, transform [A:I] into [I:A-1].
Inverse of a 3x3 matrix

The Attempt at a Solution



This is a subsection of a problem in which I am attempting to show that the set of these 3x3 matrices are a group under matrix multiplication. I was able to prove that it is well-defined, closed, an identity exists, and that associativity holds. For the inverse, it was simple to show that this set of 3x3 matrices is non-singular, but the trouble I'm running into is showing that the inverse is of the same given form so that closure still holds.

Thanks for any assistance.
 
Physics news on Phys.org
donald17 said:

Homework Statement



Given the set of 3x3 matrices of the form: [1, a, b; 0, 1, c; 0, 0, 1], where a, b, and c are any real numbers show that the inverses of these matrices are of the same given form.

Homework Equations



Using elementary row operations, transform [A:I] into [I:A-1].
Inverse of a 3x3 matrix

The Attempt at a Solution



This is a subsection of a problem in which I am attempting to show that the set of these 3x3 matrices are a group under matrix multiplication. I was able to prove that it is well-defined, closed, an identity exists, and that associativity holds. For the inverse, it was simple to show that this set of 3x3 matrices is non-singular, but the trouble I'm running into is showing that the inverse is of the same given form so that closure still holds.
What trouble are you having? Finding the inverse is straightforward, and yields the inverse in just a few steps. The inverse has the same form.
 
Actually I just solved it. Thanks.
 
Question: A clock's minute hand has length 4 and its hour hand has length 3. What is the distance between the tips at the moment when it is increasing most rapidly?(Putnam Exam Question) Answer: Making assumption that both the hands moves at constant angular velocities, the answer is ## \sqrt{7} .## But don't you think this assumption is somewhat doubtful and wrong?

Similar threads

  • · Replies 11 ·
Replies
11
Views
3K
  • · Replies 2 ·
Replies
2
Views
2K
  • · Replies 1 ·
Replies
1
Views
1K
Replies
9
Views
2K
  • · Replies 25 ·
Replies
25
Views
3K
  • · Replies 4 ·
Replies
4
Views
2K
  • · Replies 12 ·
Replies
12
Views
3K
  • · Replies 12 ·
Replies
12
Views
3K
  • · Replies 20 ·
Replies
20
Views
2K
  • · Replies 5 ·
Replies
5
Views
2K